Exercice 38

Partie

Question

Étudier la convergence simple et la convergence uniforme de la suite de fonctions suivante :

\(n \geq 1, \qquad \left\{ \begin{array}{l c l} f_{n}(x) = 0 & \textrm{si} & x \leq n \\ \\ f_{n}(x) = n^{2}(x - n) & \textrm{si} & x \in \left[n, n + \frac{1}{n} \right[ \\ \\ f_{n}(x) = n & \textrm{si} & x \geq n + \frac{1}{n} \end{array} \right.\)

Aide simple

Courbes représentatives de \(f_{1}\), \(f_{2}\), \(f_{3}\) :

Pour \(x \in \mathbb{R}\) fixé, on peut toujours trouver \(n_{x} \in \mathbb{N}^{*}\) tel que \(x < n_{x}\), par exemple \(n_{x} = E ( x ) + 1\).

Pour \(n \geq n_{x}\), \(f_{n}(x) = 0\). La suite \(\left( f_{n}(x) \right)_{n \geq 1}\) sera égale à 0 à partir d'un certain rang.

  • La convergence simple ne pose pas de problèmes.

  • Étudier la convergence uniforme sur \(\mathbb{R}\), puis distinguer les sous-ensembles majorés ou pas.

Solution détaillée

Convergence simple

Montrons que la suite de fonctions converge vers la fonction nulle. Soit \(x \in \mathbb{R}\). Posons \(n_{x} = E(x) + 1\). Alors, pour tout \(n \geq n_{x}, f_{n}(x) = 0\).

Donc : \(\forall x \in \mathbb{R}, \underset{n \rightarrow +\infty}{\textrm{lim}} f_{n}(x) = 0\).

La suite (\(f_{n}\)) converge simplement vers la fonction constante nulle.

Convergence uniforme sur \(\mathbb{R}\)

D'après les courbes représentatives des \(f_{n}\), pour tout \(n\), \(\underset{x \in \mathbb{R}}{\textrm{max}} \left\{ \left| f_{n}(x) - 0 \right| \right\} = n\). Cette quantité ne tend par vers 0 quand \(n\) tend vers \(+\infty\).

La suite (\(f_{n}\)) ne converge pas uniformément vers la fonction nulle sur \(\mathbb{R}_{+}\).

Convergence uniforme sur certains sous-ensembles de \(\mathbb{R}\)

  1. Sur des intervalles I du type \(]-\infty, b]\) ou \([a, b]\), \(a < b\).

    Il existe \(n_{b} = E(b) + 1\), tel que, pour \(n \geq n_{b}\) et pour tout \(x \in I, f_{n}(x) = 0\).

    La suite (\(f_{n}\)) converge uniformément sur tout intervalle \(]-\infty, b]\) ou \([a,b]\) vers la fonction nulle.

  2. Sur des intervalles J du type \([a, +\infty[\).

    \(\underset{x \in [a, +\infty[}{\textrm{max}} \left\{ \left| f_{n}(x) - 0\right| \right\} = n\) et \(\underset{n \rightarrow +\infty}{\textrm{lim}} \left( \underset{x \in [a, +\infty[}{\textrm{max}} \left\{ \left| f_{n}(x) - 0 \right| \right\}\right) \neq 0\)

    La suite (\(f_{n}\)) ne converge pas uniformément sur tout intervalle \([a,+\infty[\).